[Boards: 3 / a / aco / adv / an / asp / b / bant / biz / c / can / cgl / ck / cm / co / cock / d / diy / e / fa / fap / fit / fitlit / g / gd / gif / h / hc / his / hm / hr / i / ic / int / jp / k / lgbt / lit / m / mlp / mlpol / mo / mtv / mu / n / news / o / out / outsoc / p / po / pol / qa / qst / r / r9k / s / s4s / sci / soc / sp / spa / t / tg / toy / trash / trv / tv / u / v / vg / vint / vip / vp / vr / w / wg / wsg / wsr / x / y ] [Search | Free Show | Home]

/bqg/ - Bike Questions General

This is a blue board which means that it's for everybody (Safe For Work content only). If you see any adult content, please report it.

Thread replies: 378
Thread images: 70

File: bike-lane-question.jpg (27KB, 468x250px) Image search: [Google]
bike-lane-question.jpg
27KB, 468x250px
Resources:
http://www.parktool.com/blog/repair-help
http://sheldonbrown.com/
https://www.ebicycles.com/bicycle-tools/frame-sizer

Previous thread >>1093288
>>
Please look at the previous thread and see if you can answer any unanswered questions.
>>
>>1095962
Assuming this isn't a troll, you should check at >>>/n/bbg or >>1092924

I have only bought one bicycle off Craigslist but that looks like a horrible deal
>>
File: har16.jpg (22KB, 400x300px) Image search: [Google]
har16.jpg
22KB, 400x300px
Which way do you wrap your handlbars? Parktools says bottom to middle but my old man has it the opposite way so that your sweaty hands don't slip down tape.

Cloth handlebar tape if it matters.
>>
>>1097626
>not wearing gloves.
>>
>>1097628
>Your chain size will depend on how many gears you have in the back
Only in the back? I have 3 in the front as well so I thought it was front * back to get the speed.
>>
so recently i cleaned up the rust on the wheels of my bike and planning on doing some more touch ups in the other areas during the week

but some spots his the bike hard, corroding through the chrome paint, any thoughts on how to fix that or should i just get new wheels?
>>
I'm a filthy casual and need to change my chain. How can I tell if I need to change the cassette or rings as well? "See if it's worn" doesn't tell me enough.
Also what else do I need to know about doing this? Presumably I need the same number of gears unless I want to change my shifters as well.
>>
>>1097633
Yes, you choose the chain by the rear cogs.
>>
>>1097717
It's about how worn it is. The obvious answer is if you put on a new chain and get skipping, then you need a new cassette (and/or chainrings). That aside if you replaced the chain in a timely matter (say, at or around .75 on a checker), then you should be good to keep using the old drivetrain bits, if you let it get much higher than that then replacing other stuff might become necessary.
>>
>>1097717
make sure to clean your cassette real good if you don't replace it. So get a cassette tool & a chain whip to take it off.
>>
>>1097724
>if you replaced the chain in a timely matter
I'll probably need to replace them then.
How similar do I need the new parts to be? I have a Shimano 7 speed freewheel, so presumably I need another one of those (and not a cassette, which I gather is different). What do I need to look for on the chainrings?
>>
>>1097682
There isn't much you can do if the corrosion has gotten through the chrome. But if your riding on older steel rims, a new set of alloy wheels would be a worthy investment.
>>
Why do climbers grab the tops?
>>
>>1097763
aero isn't so important
If you're climbing well with a good rhythm and high cadence, you won't need to fatigue your upper body rocking side to side wasting energy
>>
File: garmin-mount.jpg (55KB, 720x480px) Image search: [Google]
garmin-mount.jpg
55KB, 720x480px
The stock garmin out front mount is pretty pug fugly, what do you guys use?
>>
Hello there, /n/iggas.
I have a chain situation. Been trying to fix it in various ways, like wiping with dry cloth, brushing thoroughly like that one guy on You Tube and even spraying with me mum's kitchen-grade de-greaser (all before applying various amounts of good [as in: working good for me prior] lube), but can't seem to get rid of faint squeaking in one link.
What do?
More de-greaser (or some other liquid, WD40 maybe?) and more toothbrushing? Just get new chain (hoped I could get away without it this season)?
Pls help and no bully, thank you!
>>
My bike is OBJECTIVELY harder to ride after I grease the chain.

What the fuck am I doing wrong? Every single time, I wipe the chain down, grease each link, spin it backwards for a good minute. For the next literally 30, 40, 50 miles (my commute is 10 miles a day, so like the first week I ride) my speed is greatly reduced and the chain actually shifts gears WORSE

Then as I go up to 100, 150, even 200 miles since greasing it (when you're supposed to grease every 100 miles) the bike runs much smoother, gears shift easier, and my speed is back up to normal

What in the the holy fuck am I doing
>>
Anybody here knows how to disassemble a Shimano RSX STI completely? The best I found online was this vid but he doesn't go far enough.
https://www.youtube.com/watch?v=LOh4zKoyJgg
>>
>>1097829
Maybe there's a pin that wasn't properly installed and that's what generating the squeak, not the chain being dirty. Check for stiff links and use a chain breaker to properly push an improperly installed pin (if there's any).
In any case, a good way to thoroughly clean a chain:
>remove chain
>put inside milk jug (or any plastic container with a wide mouth)
>pour mineral spirits in the jug (something like half a cup will do)
>shake vigorously for some seconds
>remove
>wipe
Wa la! Good as new. Remember to lube it afterwards, obviously.
>>
>>1097833
>What in the the holy fuck am I doing
Putting grease on a chain instead of oil?
>>
>>1097840
I doubt you'll find someone more thorough than RJ the Bike Guy
>>
File: isostar-hydrate-perform_5382.jpg (120KB, 532x527px) Image search: [Google]
isostar-hydrate-perform_5382.jpg
120KB, 532x527px
Redpill me on electrolytes.
I eat fuckloads of salt, probably way more than is good for me, can I really deplete my reserves on a 4-5 hour ride if it's hot outside?
>>
>>1097857
I just put a pinch of salt in my water bottles. They call them 'electrolytes' to make it seem like you can't just get them from your kitchen cabinet.
>>
>>1097829
Have you tried using oil?
>>
>>1097833
it's time to soak your chain in some wd40 and then never use grease on it ever again. just oil it or use dry lube.
>>
>>1097842
>>1097867
sorry i used grease and oil interchangably. i use the finish line dry teflon lube
>>
>>1097864
You can certainly DIY your own sports drinks on the cheap, but as long as you're not a complete retard (like going to GNC) you can order pre-mixed powders on the internet for cheap enough (compared to retail prices) that it's not really worth the trouble unless you're one of those people who gets his rocks off saving a few pennies per serving. As I see it saving a buck a serving is definitely worth the trouble but a few pennies isn't. I use some powder I got on Jet.com (ghetto version of Amazon only in the US) for like $20 that lasts me a few months per tub.
>>
I need a shorter stem for my Cyclocross. Are stems standard sizes or do I have to take mine off and measure?
>>
>>1097871
Inspect your current stem, the measurements should be written there somewhere
>>
>>1097841
>a pin
Could it get seized after, say, longer period of no use? I don't think it was squeaking when new.

>>1097866
What do you mean, oil? Like, motor oil? I don't have that shit around.
>>
>>1097875
I mean chain lubricant.
>>
>>1097871
Modern road and CX stems are reasonably sure to be 31.8 mm on the bar clamp and 28.6 on the steerer clamp. Mountainbike stems can sometimes be 35 mm. Stems for bikes older than a decade can be absolutely any wonky ass diameter.
>>
>>1097864
What I'm asking is if the salt is actually necessary at all.
>>
>>1097878
Salt comes out of your skin, so you might as well replenish it. Especially in hot weather.
>>
>>1097879
I've been salt-loading for the last 20 years and I have serious doubts I'd be able to sweat it all out in one day.
>>
>>1097882
If you eat too much salt you piss it out.
>>
>>1097876
Yes Anon, I use a dedicated bicycle chain lube that I tested myself and can vouch for (for my personal use).
And I do lube up each link individually, although don't always wait the recommended time before a ride.
Cheers.
>>
>>1097884
So all that doctor talk about water retention and blood pressure is lies?
>>
>>1097886
When it comes to salt, yes. Salt has no adverse effect on your blood pressure if you're otherwise healthy.
However, if you already have blood pressure problems, reducing salt intake helps a bit.
If you eat lots of salt, stay hydrated since you'll be peeing more to get the salt out of your system.
>>
>>1097833
>spin it backwards
No wonder, Anon. You wound it backwards. Next time spin it the right way.
>>
>>1097889
Exactly. It's like when stirring milk into tea, if you stir in the other direction you unmix the milk and tea.
>>
>>1097877
Yep it certainly was. I've ordered one which is 30mm shorter than my current so hopefully this helps with my awkward feeling ergonomics.
>>
>>1097875
>Could it get seized after, say, longer period of no use? I don't think it was squeaking when new.
Nah, that's pretty unlikely. Are you absolutely certain it's the chain that's squeaking? Some noises seem like they're coming from the drivetrain but it could very well be the fault of wheel or brake rub, some loose bolt somewhere, even dirty spokes
>>
>>1097717
This may help:
http://www.artscyclery.com/learningcenter/determiningcassettewear.html

I replace my cassette every 2 chains.
I've never had an issue with skipping or something locking up or breaking on me.

If you can't remember when the cassette was last changed, I'd just go ahead and do it this time and start fresh.

Pick one of these up:
https://www.amazon.com/Park-Tool-CC-3-2-Chain-Indica/dp/B000BR3LHQ/


>>1097775
They are all ugly.
I use a Bar Fly 4 Mini. $20 Amazon warehouse deal because the packaging was trashed.
>>
>>1097869
>i use the finish line dry teflon lube
then wtf happened to your bike? is the chain all gunked up?
>>
File: nice.jpg (211KB, 1920x1080px) Image search: [Google]
nice.jpg
211KB, 1920x1080px
What's the most underrated 80's road bike?
>>
My LBS is hiring a mechanic. They mostly cater to triathletes and freds, so the guy responded asking if I had experience with electronic installs and pressfit BBs. And it's like, I don't have any practical experience, but I know it's not that complicated, especially if you have the right tools. He wants me to call him back Monday. What do I tell him?
>>
>>1098016
I'd just bullshit about the pressfit BB, as you say it's a fairly easy thing to do and hard to fuck up with the right tools. You'll need to clarify what you mean by electronics, it could range from wiring up lights and dynos, setting up computers, to fitting and working on e-bike stuff and electronic shifting. The latter two aren't something you want to lie about as they're complicated, time consuming, and expensive if you fuck up.
>>
>>1098020
He means electronic shifting. I've seen a bunch of installation guides for di2, and it doesn't really seem any more complicated than installing mechanical shifting.
>>
>>1098023
It's not so much the installation, it's setting it up, knowing all the features, and figuring out the compatibility and component options. Tell him you know the basics of how to install it but don't have an in depth knowledge, but that you're willing to learn and able to research stuff to figure it out.
>>
V-brake and Mini-V brake pads are not interchangeable? WTF?!

>>1097894
>Are you absolutely certain it's the chain that's squeaking?
Pretty much yeah. Can't be wheel since the ""periodicity"" is too low.
Cheers.
>>
File: IMG_20170807_110634s.jpg (609KB, 1200x1236px) Image search: [Google]
IMG_20170807_110634s.jpg
609KB, 1200x1236px
I have a schwinn le tour from the 70s. the freewheel locked up in both directions. I'm not sure if its original to the bike, but the freewheel is named T.C & Sons or TCS. I cannot find any info on that brand.
What tool would i need to remove the freewheel? i have an FR-1.3 and FR-5 and they are both too large but it looks like this thing has 12 splines too.
the FR-4 on the park took website looks like it has a smaller diameter than the others and it supports "some" "schwinn approved".
I realize now I measured the wrong thing. would the FR-4 be my best bet?

tldr;

is the FR-4 my best bet to remove this broken freewheel found on an old schwinn?
>>
>>1098111
Well the FR-4 is 20 splines and only 1mm smaller so no it won't work. Are you sure the 1.3 didn't fit? You removed that locknut, right?
>>
>>1097754 here, trying o work out what I need for my front gears. I have a triple chainring with 130mm bolt circle diameter.
Can I get anything that matches that, or do I need to know about my bottom bracket as well?
There are full cranksets for the same sort of price as single gears, and I can't find any particularly cheap gears. Is there any reason not to get that? Is there an easy way to find cheap rings?
What tools do I need? I know I need a crank removal tool. How badly do I need a torque wrench? I don't even know how tight it's supposed to be.
>>
File: IMG_20170807_131714.jpg (992KB, 1920x1080px) Image search: [Google]
IMG_20170807_131714.jpg
992KB, 1920x1080px
>>1098115
It's still too small. It's nearly 2mm to small. 20.6mm vs 22.7
Maybe I'll just get a lbs to remove it
>>
>>1098111
Since I'm guessing you're gonna want to replace it anyway:
https://www.youtube.com/watch?v=WsVL1XqZve8
>>
>>1098149
the area with the splines to me looked like it was one continuous piece with the two holes.
anyway one of my LBS took it off for $5.
out of curiosity I will attempt to open up the freewheel tomorrow to take a look inside.
>>
File: IMG_3995.jpg (187KB, 1024x768px) Image search: [Google]
IMG_3995.jpg
187KB, 1024x768px
I worked at a Boy Scouts of America camp all summer teaching mountian biking merit badge to 12-17 year olds. It was a lot of fun, and I had to maintain a mix of new to very old and abused bicycles. Anyways after the season ended I was tasked with getting rid of old equipment and ordering some new stuff for next year. I'm thinking I'll be sticking with Giant Talons as our go-to for now because I already have 15 of them, but if you have any advice on tother dependable hardtail MTBs that come in child sizes in the 500$ range, I'm all ears.

I also took this old frame home with me since the hardeware is trashed. I thik I'll sandblast it, paint it and make a new commuter out of it. Any advice on cheap but dependable commuter hardware? I need some gears because my commute is in a Colorado mountain town. Pic related
>>
File: 1469922829985.png (1MB, 1072x1268px) Image search: [Google]
1469922829985.png
1MB, 1072x1268px
Is Diamondback good enough?
>>
File: IMG_20170807_170606[1].jpg (3MB, 3968x2976px) Image search: [Google]
IMG_20170807_170606[1].jpg
3MB, 3968x2976px
I recently picked up a single-speed bike at a garage sale and I feel like I need to tighten the chain a bit. However, it has pic related attached to the rear hub (derailer? I'm shit with bikes) and I have no idea how to adjust the chain tension with it there. I didn't even know single speed bikes had derailers. I would greatly appreciate any advice I could get!
>>
Hi /n/ I have a little question. I own a CAAD 8 and I want to change from 700x25 to 700x32 rims but I don't know what other things I need to change. Thanks.
>>
File: Ben10.jpg (77KB, 640x867px) Image search: [Google]
Ben10.jpg
77KB, 640x867px
>>1098271

That's a 5 speed anon, not a single speed.
>>
File: 5 speed.jpg (12KB, 283x424px) Image search: [Google]
5 speed.jpg
12KB, 283x424px
>>1098277
that's not a 5 speed, THIS is a 5 speed
>>
>>1098295

... I count 6 positions.
7, with neutral...
>>
>>1098272

Rims? Or tires?

Due to the brake calipers. The best I would think you can pull off is 700x28
>>
File: IMG_20170807_170612[1].jpg (3MB, 3968x2976px) Image search: [Google]
IMG_20170807_170612[1].jpg
3MB, 3968x2976px
>>1098277
The bike has derailers and a lot of sprockets but as far as I can tell there is no shifting mechanism. You're right though, I'm not cycling at the cadence I want because the bike is geared, and it's in a gear that's not ideal for me. For some reason I couldn't wrap my head around that until you pointed it out. Thank you anon.

Knowing that, can I move the derailer up and down the cassette without gear levers?
>>
>>1098300
your shifters are on your stem faglioli
>>
>>1098299
Rims, sorry.
are 700x28 enough for a full-holes city? Thanks :^)
>>
File: 1399313703860.jpg (60KB, 350x448px) Image search: [Google]
1399313703860.jpg
60KB, 350x448px
>>1098301
Shiiiiit THOSE are shifters? Thanks breh, I have a lot to learn about bikes.
>>
Going to start bike commuting full time year round at the end of September once I close on my house and transfer my job to one of our city locations. Commute will be 5 miles each way, though I might take a longer route back home. I got the bike sorted out with wide tires, disc brakes, full fenders, 1x9 friction shifters, proper lights and a quality u-lock.

TBC. 4chan things this is spam.
>>
Looking for recommendations on clothing that will work from 60 degrees down to 0. Not sure if I want a cycling specific backpack or a proper rack and pannier bag system. Max budget for everything is 250-300 freedom units of currency

Goddamn spam filter...
>>
>>1098334
Five miles is short enough that you can half ass proper clothing, especially if you're gonna be warm on both ends of the trip. Two medium wool sweaters (decently thick, but not so bulky that one alone restricts you) should cover your torso just fine. Your extremities are where you'll really feel it, so get some nice wool socks and gloves. Get some tights/leggings/whatever that you can wear under your pants. Maybe a hat, maybe a scarf.

You can get pretty much all of this at thrift stores, but you're gonna want to buy the tights new.
>>
i just burped/bled my rockshox reverb actuator
the seatpost really slow to go down and up.

is it possible it's caused by the fucked up bleed? i have a suspicion that i forgot to turn the adjuster to slowest in the end.
>>
>>1098304
>Shiiiiit THOSE are shifters?
jesus christ, you can see the fucking cables clamped to both derailleurs lead straight to them. you're a woman, right?

>I have a lot to learn about bikes.
obviously, since you bought a piece of shit. but whatever. you'll learn how to ride on it then upgrade. pre-emptively, I will answer your next question: read this
https://www.sheldonbrown.com/derailer-adjustment.html
then type "derailer adjustment" into youtube search
>>
>>1098486
yep that was the problem i got my shit together and had a second go at this, and now finished the burping of the actuator with the speed adjuster at slowest (open?), then the speed of travel came right back.
>>
i hear faint squealing from my shimano disk caliper (tire not rotating so it doesn't have to do anything with the rotor) when i release the brake lever.

what the hell could be it? should i grease the piston or something? it's possible it got some brake cleaner (isopropyl) on it. or it could be something else making this sound?
>>
>>1098560
The squealing is definitely coming from the caliper and not the lever? Try it with the pads and retaining pin removed as it could be them causing it, if so a light coating of something like copper grease on the back of the pads, on the pin, and anywhere on the caliper that they may be rubbing should sort it.

If it still squeals with the pads removed then the only other moving component is the pistons, you can try lubing those with some mineral oil (don't use anything else as it could damage the seals). Pump the brakes so the pistons come out and you can put the oil around the outside, then remove the fill port on the lever and push the pistons back in.
>>
The air settings guide on my Rockshox reba only suggest settings for somone up to 220. I weigh 275. What do? Set sag and go from there?
>>
>>1098570
You always ignore suggestions based on weight. Set pressure by sag, usually 20-30% based on personal preference. If the pressure can't go high enough for your weight then you'll just have to settle for more sage and if that results in you bottoming out you'll need to add some volume spacers and check your compression damping (if available) isn't too low.
>>
>>1098577
How can you tell if your shock can utilize tokens? Mine is from 2006 (Rockshox Reba SL Dual air W/motion control)
>>
>>1098578
You can't use them with that fork.
>>
File: 1500586498414.jpg (59KB, 540x359px) Image search: [Google]
1500586498414.jpg
59KB, 540x359px
my tension shifter is seized and won't move at all. is it the shifter cable that's an issue?
>>
>>1098570
i'm 260 lbs and go wigh 100 to 125 psi
>>
>>1098588
How much travel do you have? Mine is only 100mm Its my first air fork. Also, I have the negative air chamber. Any expoerience witht hat?
>>
>>1098588
sorry thats with a 30 fork reba is 32 right you should be fine with lower pressure then.
>>
>>1098589
actually my point was yiu can go lower than recommended just set a right sag.
>>
>>1098591
>>1098591
I do not have a sag ring. Just use a zip tie?
>>
>>1098592
yeah that works alright
i put one on the bike i sold
>>
>>1098566
i put my ear to both i think the caliper
thx will try
>>
Why are square taper cranks mounted diagonally on the square taper? Is there a mechanical explanation for this?
>>
>Buy seat with cutout
>Angle seat slightly downwards

>Dick still falls asleep after 8 minutes of sitting

I don't think cycling is for me.
>>
>>1098718
I don't know of many people who need to use their dick and cycle at the same time. Also try sitting on your sit bones instead of your dick.
>>
>>1098720
I'm sitting on them at the best of my ability, but I still feel slight pressure on my perineum. Only way to not feel pressure is to literally have a 90 degree angle on my back but then I can't reach the handlebars.
>>
>>1098718
Maybe your saddle is too narrow for your sitbones
>>
>>1098731
I don't think so, they're planted will inside the seat.

My proportions are pretty terrible though. I'm 184 cm, but I have a 173 cm reach and my legs are incredibly short.
>>
>>1098718
>>Buy seat with cutout
The cutout isn't for your dick, it's in case you wear jeans.
Look at a pair of Levi's. Two different seems cross paths right under your taint.
It makes a thick lump of dense cloth right under your nethers.
If you wear jeans while cycling, it's going to cause problems, but the cutout can help.
If you wear cycling shorts, you don't need the cutout at all.

>>1098722
>Only way to not feel pressure is to literally have a 90 degree angle on my back but then I can't reach the handlebars.
Go to an LBS and ask for a bike fitting.
>>
>>1098734
>The cutout isn't for your dick
no it's for your prostate and urethra
>>
File: orang2.jpg (116KB, 1280x720px) Image search: [Google]
orang2.jpg
116KB, 1280x720px
>>1098733
>>
File: idmatch3.jpg (78KB, 675x805px) Image search: [Google]
idmatch3.jpg
78KB, 675x805px
>>1098734
btw this picture should clear up the confusion about the cutouts size and purpose.
it's not about the type of pants it's about the angle of your pelvic bone on the saddle.
basically if you sit upright you don't need cutouts and for extreme aero position you also don't need cutouts as it won't change anything on a thin hard saddle.

the cutouts are important if you want a comfy seat with lot's of padding but still tend to tilt your hip forward. in this case the padding will compress into the taint area and put undue pressure and heat on it causing all kind of damage.

two solutions exist today:
1) hard saddle with little to no padding of the correct size and shape will not compress your veins and nerves.

2) cutouts on padded saddle to relieve said pressure and provide some comfort.
>>
>>1098718
post: type of seat, bike, type or riding, body-weight, back angle, seat-bone distance maybe there is an obvious mistake, other than that:
>Go to an LBS and ask for a bike fitting.
as that anon said a professional fitting might benefit you
>>
>>1098739
>>1) hard saddle with little to no padding of the correct size and shape will not compress your veins and nerves.

I tried a hard flat saddle on my road bike with some saddle to bar drop and it destroyed my taint. Surely there's some personal preference here as well.
>>
File: wo.jpg (38KB, 705x611px) Image search: [Google]
wo.jpg
38KB, 705x611px
need to buy a windbreaker for road bike riding and i need help, till now i have used regular ones and of course they suck as they are huge and stuff.
im poor and live in a far country that ruins online ordering. have these 2 options (pic related).

left is specifically made for cycling and costs 30usd, right despite not saying its for cycling has breathable armpits and has a small carry bag, costs 21usd.

should i just pay extra and go for the left one? or both are the same and the small money gap isnt justified. what do you guys think. thanks.
>>
>>1098586
Maybe. Take out the cable, and if the shifter still moves, then it's the cable.
>>
>>1098805
Well the one on the left has a hood which may come in handy.
>>
>>1098813
both have hoods, the one on the right hides in the neck, just not shown in those pics :D
>>
>>1098805
What kind of bike do you ride? If you ride with low handlebars you´ll want a jacket specially made for cycling, because the cut for regular jackets is quite awkward when sitting on a road bike. Especially when the jacket is kinda small.

If you are on a city bike or something similiar with an upright riding position you can wear normal jackets too.
>>
File: colombier.gif (13KB, 1136x673px) Image search: [Google]
colombier.gif
13KB, 1136x673px
Hello. This is more of a general bike related question but here I go

I live in the flatlands but I want to train for long cols in the alps and the like. I would like to get a hometrainer so I can train at home and pretend to ride up a mountain by having to overcome the wattages referencing the actual mountain profile

is this possible? any such devices?
>>
>>1098743
>Surely there's some personal preference here as well.
or the size was wrong. i used hard saddles so far, and they only destroy my sitting bones. maybe you have real fucking wide pubic arch.

as a general rule harder the saddle the less it compresses on the wrong places and the longer you can ride it.
>>
>>1098847
A smart trainer can do that, but they're expensive. I don't have one myself but my dad has a Wahoo Kickr and he really loves it.
>>
'97 Marzocchi Z2 crown has a crack at the bolts... any idea where I can get a replacement / aftermarket one? I found some for XC 500, so there are people making this stuff.
>>
>>1098885
I highly doubt people are making replacement crowns, if they were they'd cost more than just buying another fork. You may be able to find an old replacement but I wouldn't hold your breath. Either replace the fork or if you have some sort of emotional attachment to it get it welded.
>>
>>1098847
https://www.youtube.com/watch?v=vVLls05qzV4
>>
>>1098847
http://road.cc/content/buyers-guide/217308-15-best-turbo-trainers-and-rollers-%E2%80%94-smart-and-traditional-home-trainers

Yes. Long story short; you want Zwift.
>>
>>1098870
>>1098920
>>1098936
Thanks guys, ordered a Wahoo Kickr Snap
>>
what are some good books/magazines on biking?
>>
I was on my rollers for 10 minutes or so and I had to stop for a bit but when I got back on them I kept feeling this bump and when I checked the tire had a bulge like an S.
Time to just replace it? How can I make sure this doesn't happen again?
>>
File: q1bihzih1bez.jpg (50KB, 720x720px) Image search: [Google]
q1bihzih1bez.jpg
50KB, 720x720px
Could I get away with a diet of mostly Chicken, Rice, Broccoli and Sandwiches? I don't eat enough as it is and I abhor everything to do with food. Prep, clean up and actually eating but I'm fucking tired of being hungry all the time and not having energy on my rides after work (12 hour shift).

In 12 hours (while at work) my meals would be something like

>PB&J (Local jellies and all natural PB)
>1 piece of chicken, rice and Broccoli with maybe gravy or some other sauce
>PB&J
>1 piece of chicken, rice and Broccoli with maybe gravy or some other sauce

I could maybe do 3 meals of chicken and two sandwiches but I'm not sure yet. I work 12 hour shifts and usually ride as soon as I get home (8PM/AM) so getting food ready for the next day but also being fueld is important

I like the chicken route becasue I can prep it in multiple different ways, it's a quick turn around to get meals ready for the next day and it's tasty, healthy and filling. I could even add some flax seed oil if necessary.


I know a lot of body builders use this diet so I can't see why it wouldn;t work for cycling.
>>
>>1099251
Replace jelly with fruit and chicken with vegan burgers and beans, then u got a healthy diet
>>
>>1099251
that sounds awful
>>
>>1099253
I would be eating a few bananas a day. I see beans being filling but not the vegan burgers would have to prob eat 2-3 per meal and they're much more expensive but I may give it a try.

>>1099254
Not sure what sounds awful about it. It's quick and easy. My dissatisfaction for food prep, clean up and eating goes so far I would rather go hungry than deal with but obviously with cycling I can't do that.
>>
>>1099251
Wide and varied diet is very important. It sounds like the solution is some sort of 50s gender role ricenigger waifu.
>>
>>1099251
I'm with the other guy, get some more fruit (banans are gud) and get some beans.
They'll assblast you to higher speeds, scientistically prooven dude, but really they are a wonderful pairing with rice.

Maybe some more leafy greens? Idk dude, BBs do a lot of dumb shit when it comes to diet just because they need to gain or keep their gains and that boring as diet will make your rides boring too, like I mean, having a yummy snack on your ride is a nice thing.
>>
>>1099257
I would be eating different things on my days off, this is just the diet for when I work. I work 3 days on, 3 off for two cycles then do the same on nights and my shifts are 12 hours. Coupled with cycling after work I need something quick, that I can prep batches of and is decently healthy and will give me fuel.
>>
>>1099228
I had this too, once although from riding outside. When I looked at the inside of the tire I saw that the beading had come loose a bit and I could see the wires, which meant if fully inflated it couldn't hold the original shape together. I replaced it and upgraded with a better tire as it was a fairly low quality tire
>>
>>1099257
There's no variety in always eating a varied diet.
>>
File: photo_2017-08-11_14-42-20.jpg (118KB, 960x1280px) Image search: [Google]
photo_2017-08-11_14-42-20.jpg
118KB, 960x1280px
can you guys tell me if this is a bb30 bb ?
there's a tigara crank on there and the website says it's pressfit.

I'm a bit confused here
>>
>>1099351
Well, it's definitely not a threaded one, there'd be a bit on the outside of the frame.
>>
i live in a decently small town. i want to ride a bike to the golf course, which is less than a mile away. i want to maybe ride a couple miles to the store to pick some stuff up. i sweat a lot, so i don't think i could ride to work every day.

what's a decent and inexpensive bike that is good for this kind of short distance errand running?
>>
>>1097857
kalium (potassium) is also important aside natrium (sodium) if you sweat and drink a lot. drinking in excess of 3 liters of clear water is ill advised. if you only drink like a liter of water during your training it doesn't fucking matter. 6l of clear water can kill you so if you are intensely training and drinking and sweating a lot you need the fucking electrolytes and salt is not enough.
>>
>>1099373
90s rigid MTB
>>
File: 1496717055984.jpg (21KB, 326x339px) Image search: [Google]
1496717055984.jpg
21KB, 326x339px
r8
>>
>>1099371

yeah ,but are there 24mm press fit bbs ?
or is it a bb30 with a 24mm spindle ?
I really don't get it.
>>
hey guys whats an affordable starter bike for a road bike I would like to get into competing
>>
>>1099386
All hollowtech II cranks have the same diameter spindle as far as I know.

>>1099389
Depends what your budget range is. For 1000-1500 you could probably get a nice new bike with tiagra, alu frame and carbon forks.
>>
>>1099386
Yes, Shimano makes press fit bottom brackets now. If you don't want to mess with that shit when it comes time to replace the bottom bracket you can get press fit to threaded converters.
>>
>>1099389
For competing? At what level?
What's your budget?
You can spend $1500, but you're getting marginal improvements over a solid bike that's half that price. I recommend getting a $750 alu bike with rim brakes and Tiagra if you can get it, but going down a notch to Sora is PERFECTLY FINE.
You want something you can train on more than something you will race on.
$100 of aero clothing will do you better than $100 of bike at almost any level.
>>
>>1099394

but could I justput a normal press fit bb in an run a bb30 cranks ?
I know that shit gest a lot of hate , but I'm a sucker for ebay deals and you get some pretty bling crank for really low prices if they're bb30
>>
>>1099400
If that's what you want to do then sure, go right ahead. It's true that there are a lot of cheap BB30 options on Ebay but you're not gonna gain much from switching to them from what you have now. Basically you'd be dropping about of weight with some carbon cranks from SRAM, personally I wouldn't say it's worth the cost but it's up to you.
>>
>>1099260
>beading had come loose a bit and I could see the wires
That's what I just looked at and mine was the same, cool mate, I'll just go get another tire today. Thanks dude.
>>
File: virp46-1.jpg (107KB, 1200x800px) Image search: [Google]
virp46-1.jpg
107KB, 1200x800px
Has anyone here tried these?
Good/bad?
>>
>>1099405
According to the BRR test they're mediocre rolling resistance, and ok puncture resistance for a road tyre. I'd personally spend a bit more money.
>>
File: IMG_4016.jpg (3MB, 4032x3024px) Image search: [Google]
IMG_4016.jpg
3MB, 4032x3024px
I have a question for accomplished saddle hunters-

Just got a new bike, haven't ridden seriously for 5 years, and the saddle fucking killed me. did about 20 miles on it the first day, maybe 25 the second, today I couldn't take it anymore and took out a demo saddle- pic related

It's immediately better.

Should I buy it? It's so much better than the ultra lUskinny one that came stock. Or is my worth trying more saddles?
>>
Autismo question -

How do people do that thing where they mount/dismount the bicycle by stepping on one pedal whilst the bicycle is moving?

I tried to do it and I keep falling off lol
>>
>>1099426
How else do you get on the bike? Do you hold onto something so you don't fall over as you put both feet on the pedals?

It's pretty damn simple, put one foot on one of the pedals and push off with the other like a kick scooter, then swing the other leg over the bike and put the other foot on the pedal. Alternatively if you have the first pedal in the right position as you step on it it'll propel the bike forwards so you don't need to push off.
>>
>>1099433
>How else do you get on the bike
I sit on the saddle and pedal off

What leg am I supposed to put on, which do I swing? this honestly doesnt go into my brain
>>
>>1099434
Whichever leg you want, try it both ways and see which is easiest.
>>
>>1099434
Stand next to your bike (for this example let's assume you're on its LEFT)
Put both hands on the handlebar
Put your LEFT foot on your LEFT pedal (crossing your left leg in front of your right)
Push yourself with your RIGHT leg so you start slowly moving
Swing your RIGHT leg over the saddle and into the RIGHT pedal
For dismounting, do the same but backwards.
>>
>>1099433
>>1099438
or stand above your bike so that the frame is in between your tighs, put one pedal forward and place foot on it as you sit on the saddle, while keeping yourself stable with the other foot. push down on the pedal to get started and lift the other leg and place on the other pedal. that's one way to do it also.

when i have to start on a climb it's pretty much the only way.
>>
>>1099402

great thanks !
I'm not really looking for anything but If I find something really nice I can consider it.

I know it's stupid but I'm really into upgrading stuff.
it's kind of a purpose in itself.
>>
>>1099441
He was specifically asking about the "cowboy straddle" though
>>
>>1099438
thanks

>>1099441
>when i have to start on a climb
also useful thanks
>>
I wanted a bike that wouldn't lose balance running over potholes. The guy at the bikeshop suggested a mountain bike, so I got a cheap mountain bike for $500. Have I been meme'd? It rides nicely, but I don't know much about bicycles. Will I have problems with it down the line? Are there better options available?
>>
>>1099539
Maybe if you told us what bike you actually bought
>>
>>1099547
Jamis x26, it's a kids bike (I'm 5'5" lol)
>>
File: IMG_1711.png (329KB, 1899x761px) Image search: [Google]
IMG_1711.png
329KB, 1899x761px
Is this a good beginner fixie crankset? i want quality but also affordable. what do you guys think? Any recommendations?

https://www.ribblecycles.co.uk/stronglight-track-2000-jis-chainset/#pid=20806
>>
>>1099539
>I wanted a bike that wouldn't lose balance running over potholes. The guy at the bikeshop suggested a mountain bike, so I got a cheap mountain bike for $500. Have I been meme'd?

Yes and no.
It's actually pretty complicated.

All that really matters is that you ride a lot and enjoy it. If you get meme'd a few times along your personal journey of discovery, that's okay.
>>
>>1099560

>CLEARANCE
>STRONGLIGHTSTRONGLIGHTSTRONGLIGHTSTRONGLIGHT
>STRONGLIGHTSTRONGLIGHTSTRONGLIGHT
>STRONGLIGHTSTRONGLIGHT
>STRONGLIGHT

Either you are trolling or you have a severe mental handicap.
>>
>>1099560
ignore this retard >>1099565 it is actually a good beginner crankset
>>
>>1099573

>$90 for a trash-tier fixie crank

No. Not it is not.
>>
>>1099561
Can you tell me anything more specific?
>>
File: 8167352_orig.jpg (19KB, 254x194px) Image search: [Google]
8167352_orig.jpg
19KB, 254x194px
>>1099392
>>1099396
thank you for advice!
>>
>>1099426
I don't see it too often, mostly old people with 20kg single speed town bikes do it. I think I've tried doing it once or twice, but I don't get the benefit.

>>1099433
1. swing your leg over the bike
2. start pedaling
3. ???
4. now you are successfully riding your bicycle

>>1099434
oh god please don't continue to try it, if you can't figure this out yourself.

>>1099565
absolute shitposting

>>1099586
It sounds like you have a working bicycle and you like it. Surely there are faster, lighter, fancier bikes out there, but who cares?
Try making a more specific question, if you want a specific answer.
>>
File: bolttypes500.jpg (53KB, 500x375px) Image search: [Google]
bolttypes500.jpg
53KB, 500x375px
Long shot, but is it in any way possible to use brake calipers with recessed mounting on a frame built for traditional style brake calipers?
>>
File: yho9pgM.jpg (202KB, 1600x1200px) Image search: [Google]
yho9pgM.jpg
202KB, 1600x1200px
How do I ride a road geometry bike without getting my dick crushed on the saddle?
>>
>>1099646
set your bike up properly
wear padded bike shorts
find a seat that fits
>>
>>1099644
Yes, just drill the rear of the fork so the nut can fit through.
>>
>>1099646
nobody told you?
the first step to getting aero is having your dick removed.
it's honestly worth the gains.
>>
>>1099675
That's what I was trying to avoid, thanks
>>
File: cogsucker.png (695KB, 730x974px) Image search: [Google]
cogsucker.png
695KB, 730x974px
so when im biking the bike has trouble shifting (gripshift) from the 6th to the 7th gear (the smallest cog), but when I have it upside down, it works like a charm

any ideas?
>>
File: prod82431_IMGSET.jpg (29KB, 500x505px) Image search: [Google]
prod82431_IMGSET.jpg
29KB, 500x505px
>DH Derailleurs on City Bikes
I'm having trouble figuring out why something like pic related isn't BIS for a 1x10 city commuter.

>Short cage makes for snappy shifts
>Clutch keeps chain on and reduces chain slap
>Literally designed to be smashed into rocks and keep going

The only drawback I can find is weight.
>nb4 general Shimano hate

Convince me that there's a better option for a year-round city commuter.
>>
>>1099447
that's not very safe to do with lose pants tho.
>>
Im a retard who doesnt know anything about bikes. what bike should I buy to travel around town?

>>1099251
/fit/ here. bbers eat that for cutting. its very low calorie. you want something high calorie. Ive struggled with low appetite + long work hours before too. my suggestions would be
>whole milk
>eggs
>adding olive oil/coconut oil
>>
>>1099693
Why? It's a simple and easy thing to do.

>>1099695
Something like that would make me think the B tension needs adjusting, but that derailer probably doesn't have that adjustment. Does the derailer look bent to either side at all? Everything should be pretty much perfectly straight, bends are likely to happen at that black plate that it held on by the axle nut and the pulley assembly that holds the two toothed wheels.

>>1099704
It doesn't need to be a DH derailer, just a MTB one. If the cage is too short it won't be able to take up the chain growth from a wide range cassette and double/triple, DH ones are designed for close ratio 7 speed cassettes and a single chainring.

>Literally designed to be smashed into rocks and keep going
No they're not. The attachment link to the hanger is the same as any other Shadow+ derailer and will be the first thing to bend. The only difference between that and a regular MTB derailer will be the shorter cage (greater clearance from obstacles) and the parallelogram might be a tiny bit beefier but those very rarely bend anyway.

But yeah, if you have flat bar controls a Shadow+ MTB derailer wouldn't be a bad choice for a "city bike".
>>
>>1099715
>and the parallelogram might be a tiny bit beefier
it's beefier to hold more tension i think not to resist crashes.
>>
>>1099705
But why would anyone ride with lose pants though? It's like swimming with shoes
>>
>>1099720
well i do, because that's a pants i bought for workouts year sago and it's a pretty good one made by adidas it airs very well. only thing is it gets stuck in the saddle all the fucking time when i try to get on. but once i'm on it doesn't bother me one bit.
>>
>>1099715
>Something like that would make me think the B tension needs adjusting, but that derailer probably doesn't have that adjustment. Does the derailer look bent to either side at all? Everything should be pretty much perfectly straight, bends are likely to happen at that black plate that it held on by the axle nut and the pulley assembly that holds the two toothed wheels.

perfectly straight, fucking weird
>>
>>1099729
With the bike right side up does it shift better if you apply an upwards force at the derailer pivot (where it attaches to the black plate)? That's where the B adjustment is on other derailers, could be that the spring in that pivot is just worn out.
>>
>>1099695
>>1099729
did you check chain stretch?
>>
>>1099734
ill check that tomorrow, thanks man

>could be that the spring in that pivot is just worn out.
i actually thought about that, since it shifts when its upside down but has trouble doing it normally

>>1099735
no sir, but ill do it tomorrow, the wife wants me to clean up the bike mess
>>
File: avidpost.jpg (78KB, 944x668px) Image search: [Google]
avidpost.jpg
78KB, 944x668px
Going from 180 to 160 rototrs, fork is post mount 160. Is 17mm the correct size for avid elixir calipers The bolts that were on the 180 w/ adapter are too long.
>>
>>1099745
17mm thread length? That should be alright, I doubt 12mm would bottom out in the fork (the caliper is about 5mm thick) and that's plenty of thread engagement. You could always use the longer bolts to check, measure the thread length and then bottom them out in the fork and measure how much is exposed. If you're using those angle adjustment washers that SRAM likes to use then don't forget to account for those.

Why you going from 180 to 160 anyway? Weight weenie?
>>
File: chinkrotor.jpg (65KB, 603x492px) Image search: [Google]
chinkrotor.jpg
65KB, 603x492px
>>1099748
I'm gonna use 1 set of the washers instead of 2 per the avid pdfs I could find. I bought the 17mm bolts, hopefully that will work. Going to 160 because i got a good deal, the HS1s i have now squeal like a bitch,weight savings, comes in colors, and I only weigh 135 lbs so I doubt i'll notice a diff.
>>
>>1099754
So only the washers between the fork and caliper, not the ones between the caliper and bolt head? Just know that if you do that you'll have no angle adjustment but you shouldn't need it anyway, no idea why SRAM feels the need to include it.

You may also be able to ditch the inner washer but I'm not sure how that affects the spacing, whether it's compensated at the caliper by having the tabs moved outwards or if the adapter is shorter than one for a caliper that doesn't use the washers.
>>
how the hell do i find out what rotor size my frame is rated for?
>>
File: carbon rotors.jpg (194KB, 717x480px) Image search: [Google]
carbon rotors.jpg
194KB, 717x480px
>>1099768
google it- but you can prob use 160-203 front and back with adapters.
>>
>>1099768
The manufacturer's site will usually say.
>>
>>1099771
i wish, it's a grand canyon al hardtail i know the fork is rated for 180mm because sram is not retarded but the rear i can find no info on not even in the fucking manual. i'm currently rolling 180s front and rear because the 160 overheated way too easily.
>>
>>1099775
According to Canyon "Our bikes are approved for use only with brake disc diameters which are factory fitted."

https://www.canyon.com/en-gb/service/supportcenter/article.html?supportcenter_articles_id=198

That said going 20mm over the stock/native size will almost always be okay. I wouldn't go +40mm if you're particularly heavy, riding hard, or it's a light use XC/CX/road frame.

There are also other ways to minimise heat such as pad material, finned pads, and finned rotors with aluminium cores.
>>
File: 20170720_171116.jpg (669KB, 2064x1161px) Image search: [Google]
20170720_171116.jpg
669KB, 2064x1161px
>>1099781
> if you're particularly heavy, riding hard, or it's a light use XC/CX/road frame.
well fugg
>finned rotors with aluminium cores
yeah i got those babies

the thing is normally i wouldn't exert much force on the rear brake it's more about thermal endurance on long down rides. but i started to practice hops and stuff like that and when you put the bike around and you press the brake and rear wheel touches ground it could pull with pretty significant force. i think i have to watch out for that.
>>
I just adjusted my rebound all the way + and now it has a "piston sound".

Is this a sign of putting too much stress on the fork?
>>
>>1099803
Did you turn it so it's fastest or slowest? Different manufacturers have different meanings for +, some have it mean the most rebound damping (so slowest rebound) where as others have it mean the highest rebound speed (least damping).

If you have it set slow then you may be noticing the noise of the oil travelling through the holes in the piston in the damper, it's sort of a squelching noise. If you have it set fast then the noise could be caused by the fork topping out and hitting the internal bumper.
>>
>>1099804
I think it's on it's slowest. Sounds exactly like a bunch of oil being forces through a small hole (which would make sense because that's what's happening, right?)

Main question is it dangerous to my fork?
>>
>>1099805
Nah it's not dangerous but the majority of forks have an unnecessary amount of rebound damping at the full on end. If you ride it like that the fork is just going to pack up (not extend back fast enough) and you won't be able to use all of the travel.
>>
>>1099809
I think that's fine. Just playing around with it.

It seems much better over roots and small obstacles. Does that make any sense?
>>
>>1099810
Over small stuff it's going to feel better as the damper is preventing the spring from pushing back at you, but like I say you'll eventually be sitting so far down in the travel that the fork is going to become hard (spring rate increases as it compresses) and it's going to feel worse and you'll lose traction.

You want the rebound to be quite fast, as fast as possible without it working against you. If you stand on the bike and push down hard and quick on the bars you want that front end to come up as quick as it can without lifting the wheel off the ground or bucking you backwards.
>>
>>1099810
Yeah and no.
More dampening equals more comfort but more energy loss thus slower ride.
Also you have to rely less on thd front brake if you set slow rebound and low psi because of the packing up.

Everyone has to find the settings that match his style. All has pros and cons and speed and comfort will have to compromise between.
>>
>>1099813
hmm I think I'll just have to play around with it. It was somewhere in the middle. The only other bike I had with suspension was a hardtail Norco Charger, where the geometry is right over the front. It didn't have rebound control.

What are the advantages of both settings (high rebound and low rebound). I'm probably just blaming my bike for my shortcomings, but I don't think I liked it in the middle.

The terrain I'm on is pretty diverse but it's mostly smooth with small rocks and roots.
>>
>>1099817
i personally prefer slow rebound and actively managing availabld travel. for example brake before you hit a rough section and let go of the brake while easing up on the bar to regain max travel.
>>
>>1099817
Slower rebound is going to give more comfort over small and infrequent bumps, as you've discovered. The disadvantage being it will pack up over successive bumps and you'll have reduced traction (as the wheel won't extend downwards and into the ground quick enough after hitting a bump).

If you imagine a theoretical ideal setup the bars would be staying at the same height and the wheel would move up and down to follow the terrain perfectly. Not going to happen in the real world but that's what you're aiming for, you want that rebound pushing the wheel downwards as soon as possible after hitting a bump, not so fast that it rebound before clearing the bump and not slow enough that the bars sink downwards.

1:40 is a pretty good example, as they go around the bumpy berm
https://www.youtube.com/watch?v=u8Y-yQ_TJKI
>>
>>1099827
Cool
>>1099820
I think I'mma just stick to the low rebound for a while. More than often it's just 1 big bump or a small rock garden so I think if it pushes out slower I wont really notice.
>>
>Go out for standard ride (20+ miles)
>Don't take protein shake after ride
>Legs sore the next day
--
>Take protein after ride
>Legs not sore

Is this normal?
>>
>>1099833
Yes, if your nutrition is shit
>>
>>1099833
Protein after a workout helps prevent/diminish DOMS so yeah, I mean you could also eat but a shake is easy to drink and it's absorbed more quickly than solid food.
>>
>>1099833
I once didn't take any for 24 hours and every muscle in my body started to hurt like I just worked out.
>>
>>1099449
new to the convo here

just going to mention that if you currently mount the saddle and then pedal to start as you stated, then your saddle is WAY TOO LOW in terms of getting anything close to an efficient use of your legs. It may be comfy, but you're wasting a lot of energy. Once you learn how to mount the bike properly, raise your saddle until your leg is a bit shy of fully extended at the bottom of the crank. If you're new you don't have to do this all at once, but as you get better and go longer distances, you'll find this arrangement more rewarding.
>>
>>1099864
learn human and bike anatomy please before you post bullshit!
you do not get on the saddle this way with the pedal all the way down but forward ag roughly 90 degrees this gives you an easy clearance from the saddle and you can get more by extending your feet. all in all you can get 20 to 25cm clearance above a perfectly by the book set saddle when mounting.
>>
File: 1502046569382.png (202KB, 957x891px) Image search: [Google]
1502046569382.png
202KB, 957x891px
Why do vintage bikes not have any high gears?

I bought a late 80s road bike, it has 12 gears. Which would be fine if they were spaced out. But most of the gears are low end for speed on sprints and there are no gears for climbing.

Did people in the 80s just not go up hills?
>>
>>1099945
Spinning to win only really became popular relatively recently, people used to just grind and mash up hills.
>>
File: 4 (1).jpg (190KB, 1920x1080px) Image search: [Google]
4 (1).jpg
190KB, 1920x1080px
Is there more to removing a crank from bike than just releasing the bolt in the middle of the crank?

I'm going to be taking a bike apart soon for restoration, I had a go at removing the bolt just a second ago to see if it would be easy to take apart, but even with the bolt removed the crank arms were still solidly attached
>>
>>1099945
Back then, as now, freds wanted to use what the pros used, which meant 52-55t on the big ring, 39-44 on the small, and 11-19, 12-19, or 12-23 in the rear. More than 23 was strictly haram then. These days you're seeing way more forgiving gearing because mash2win is dead (Lance and his thing where he'd use 14-23 corncobs and spin at 120 up hills played a big role in that, I imagine), and because you can have wide ranges without big jumps between sprockets.
>>
>>1099954
Having just done this yesterday: yes. The cranks are held on very firmly by friction, and you need a crank extractor that screws into the crank itself, and has another screw that extends in against the bike to push the crank off.
That's probably not very clear, but look up "crank extractor".
>>
>>1099407
read literally the first 4 paragraphs of sheldon on saddles www.sheldonbrown.com/saddles.html
>>
>>1098296
>chain slips off of drivetrain
>"i'm in neutral"
>>
File: IMG_20170727_131510.jpg (2MB, 4160x2580px) Image search: [Google]
IMG_20170727_131510.jpg
2MB, 4160x2580px
>>1098300
how much this vintage pug cost you man?

Here's mine I bought recently.
>>
Hello m8s I'm trying to shift from 700x25 to 700x32 but I don't know if I need to replace only the tire or the rim too. Thanks <3
>>
>>1099989
just the tyre
>>
File: IMG_0740.png (577KB, 1536x2048px) Image search: [Google]
IMG_0740.png
577KB, 1536x2048px
Are these good wheels? I need to replace the crappy 24 spoke wheels my bike came with
>>
>>1099989
What bike is it? Some caliper brakes won't fit a tyre that wide under it.
>>
>>1099704
Do it! I'm considering putting Zee on my CX bike at some point (it's currently set up as singlespeed).
>>1099715
>DH ones are designed for close ratio 7 speed cassettes and a single chainring.
Shimano Zee (the one pictured) comes in both close ratio and wide ratio versions. And all modern day DH derailleurs are at least 10-speed.
>>
>>1099967
wow nice the video describes exactly what i have been talking about here: >>1099927
>>
>>1100000
Sure, but then you have a mid cage instead of a short cage, making it no different to a Deore/SLX/XT/XTR with a mid cage. A mid cage may not even be enough for commuter gearing, not read much into it as I don't fuck with doubles or triples.

If you're get Zee just because it's Zee and not for the DH specific features then you're an idiot. If you get one because it ends up being cheaper and easier to find than a mid cage Deore/SLX then that's fine.
>>
>>1100011
Both Zees have a short cage, the only difference for close ratio one is that the pulley wheel is closer to the cassette. Basically the derailleur is identical, only the connecting link differs. I'll use Zee, only because I have it leftover from my freeride bike, but yeah, all Shadow+ derailleurs should be fine.
>>
>>1100020
Well it seems like you're right, the Saint model actually comes with an adjustable link to switch from close to wide ratio. In that case it's even less likely that it'll be able to take up the chain slack from a double or triple on a commuter bike, but I'll gladly be proved wrong.
>>
So im finishing my fixie build but i need to accommodate my gear ratio for my commute. how many gear inches are ideal for a place that is flat but has small and a few large hills? i live in the north Atlanta suburbs
>>
>>1100049
There's no such gear ratio
>>
>>1100049
>many gear inches are ideal for a place that is flat but has small and a few large hills
If it was me I'd 42x17 for the skid patches, if you skid, and to spin up them hills.
Idk how you roll but you'll have to compromise on top speed or mash up hills.

If you have a flip flop hub then your options expand
>>
>>1100051
>If you have a flip flop hub then your options expand
>Hold on, just gotta remove my wheel, flip it over, and re-tension my chain so I can climb this hill
Fixed is retarded unless you ride on a track, play bike polo, or perform ballet. Single speed is retarded if you ride terrain that would greatly benefit from multiple speeds. Skid patches should not be something that you should be concerned about. The choice between mashing or going slow is not one you should have to make.
>>
>>1099833
>>1099839
>>
>>1099946
>>1099963
Has more to do with more cassette gears = smaller steps = wider range possibilities, but yeah sure; let's just call it a cultural thing.
>>
>>1099833
it's psychological.
protein doesn't help with soreness it's caused by inflammation of the muscle from lactic acid and damage to the muscle tissue.
protein just helps to regenerate and can help to prevent muscle loss during long endurance workout when you bottom out on sugars.
>>
>>1099927
what you're claiming I said and what I actually said are two different things.

I said nothing about getting IN the saddle, I said that if he can sit in the saddle before he's in motion--which is what he said--then obviously his feet are both on the ground and therefore his saddle is too low. My post was about proper seat-height, not what to do with the cranks when you mount--other anons already explained that. Re-read his posts, the replies, and what I wrote, and go be mad somewhere else.
>>
>>1099646
Slide the saddle forward and tip the nose down. Haters gonna hate, tell you about KOPS and shit, but as it happens, UCI relaxed their rules and a lot of guys jumped to do this in the races too.

ps. I did it way before I knew there were rules about saddle placement. I prefer not sitting on my dick, but still having my handlebar low.

>>1099654
I don't get how the padded shorts are going to help anything. If your bodyweight is carried by your genitals, it's going to be very painful regardless of a few mm padding.

>>1099864
Eh, I can stand tip toes in the saddle of my road bike. I often stop at lights with one foot clipped in and just touch the ground with the other foot, while sitting in the saddle. If the saddle was any higher, I'd have to ride with my ankles extended too...

>>1100011
The longer cage Zee is specced for 11-36 cassette IIRC. I'd imagine that is enough for most uses.
Also I'd guess it would be pretty safe to assume Zee has a stiffer spring than the XC-style RDs.

>>1100051
>42x17
>spin up them hills
rip knees once again
>>
>>1100096
>I said that if he can sit in the saddle before he's in motion--which is what he said--then obviously his feet are both on the ground
Why must both feet be on the ground?
>>
>>1100098
>The longer cage Zee is specced for 11-36 cassette IIRC. I'd imagine that is enough for most uses.
As me and the other guy cleared up, the wide ratio Zee doesn't have a longer cage. Whilst it can take up the chain slack going from 11 to 36 in the rear it may not be able to take up the slack of a double or triple crankset, something that most commuter bikes have.

>Also I'd guess it would be pretty safe to assume Zee has a stiffer spring than the XC-style RDs.
They're not XC, just not as optimised for DH. A stiffer spring doesn't offer any advantage on a commuter bike either, it just makes shifting harder.
>>
>>1100096
>put one pedal forward and place foot on it as you sit on the saddle
you mean this part? it's pretty obvious you sit on the saddle by standing on the pedal.
sequence is:
>stand above bike
>put pedal forward maybe a little higher and place a foot on it
>stand on the pedal and extend your leg and sit on the saddle in one smooth motion

>his feet are both on the ground and therefore his saddle is too low
i have my saddle in perfect height and i can put both feet on the ground when i stop. there is a trick to it tho.
>>
>>1100102
Fair enough, seems that there were a few guys wanting different things;
>>1099704
>BIS for a 1x10 city commuter.
He wanted a 1x10 commuter, I think the 11-36 is pretty good for that.
>>1100000
>I'm considering putting Zee on my CX
He would benefit from the stiffer spring. He'd have to run 1x too, to make it work, but that's not rare in CX.

Personally, my interest in the Zee was based on the idea that my chain wouldn't be all over the place on my MTB, but it seems I'm better off saving for something with a clutch.
>>
>>1100109
My mistake, I must've glossed over where he said 1x10.

For the CX I'd say the stiffer spring still isn't advantageous as it won't see terrain as rough as a mountain bike, never mind a DH bike, and they do just fine with Deore/SLX/etc. Also he's going to be using either brifters, barcons, of downtube shifters, not sure how easy they'd be to use with the increased tension.

> my chain wouldn't be all over the place on my MTB, but it seems I'm better off saving for something with a clutch.
A clutched derailer (anything Shadow+) would help a lot with that, more so combined with a narrow wide chainring if you run 1x(you could even use a chain guide as well but that's probably overkill). I used to drop chains quite a bit until I got a chain guide but I still had a lot of chainslap, then I removed the guide and got a clutched derailer and a narrow wide and I've only dropped the chain once and that's when I forgot to turn the clutch back on.
>>
You guys think Canyon is gonna announce USA release this week or what? Gotta get me hands on one of those Aeroads before the leafs fall
>>
>>1100090
Even with smaller cassettes pros weren't ever using compact cranks or anything like that, were they?
>>
>>1100142
I don't think compact cranks existed back in the day, at least nothing racing oriented.
>>
>>1100142
In that era serious roadies in europe would ride triples
>>
Anyone tried this saddle? My gooch kills me every time I ride long distances. is this comfy or meme?
>>
>>1100180
Go to LBS, try some saddles. They'll let you try out saddles.
>>
File: ouch!.png (58KB, 1294x669px) Image search: [Google]
ouch!.png
58KB, 1294x669px
>>1100153
For the most part pros would avoid triples because of q-factor and chainline reasons. Instead, they'd do weird stuff like run 53/34 if there were a big climb at the finish where they'd not have to shift back up to the big ring, sometimes go up to 25t on the back. That would happen in Italy and Spain, though, where some of the climbs are insane. And since everyone started moving to lower gearing, they've added more insane climbs like Zonclon, where a lot of the riders run close to 1:1 for their lowest gear.

It would be really weird to see a pro roadie do something like run a sub compact chainset, although with races that involve gravel climbing sections becoming more popular, who knows?
>>
File: avid.jpg (9KB, 246x205px) Image search: [Google]
avid.jpg
9KB, 246x205px
>>1099748
it needed a 29mm- after i spent $12 on the wrong bolts i gotta spend $8 more- no wonder ppl hate avid.
>>
File: 20170615_100557[1].jpg (727KB, 2000x1500px) Image search: [Google]
20170615_100557[1].jpg
727KB, 2000x1500px
>>1100185

Me and a few friends are going to attack that climb once the weather cools down a bit.

I'll be using 50/34 and 11-32 cassette because I'm not a pro.
>>
File: Selle Italia Max SLR Gel Flow.jpg (245KB, 1000x1000px) Image search: [Google]
Selle Italia Max SLR Gel Flow.jpg
245KB, 1000x1000px
>>1100180
I have a step down from that, and have no issues with it.

If its uncomfortable, then try a different seat.
>>
>>1100292
this is the seat i want to buy online but lbs only lets you try out seats if you end up buying from them. they sell with 30% prem.
>>
>>1100280
Contador has used 36x32 on Zoncolan so I'm certain that many pros have done 34x32 on that climb. The gradient is 22% at one point. Insane. And I'm so jelly of you.
>>
>>1100294
order online
wrap the rails with tape, try it out.
>>
>>1100295
How can you even achieve such small chainrings? MTB crankset? The q factor would be shit.
>>
>>1100312
36/34 is the chainring, not 32.
>>
>>1100294
Wtf? Just try them then walk the fuck out. What are they gonna do, call the cops?
>>
Will a patched tube hold 140psi?
>>
>>1100323
Should do, I'd be more worried about the tyre and rim. Don't know why you'd even want to run that high.
>>
File: IMAG0260.jpg (3MB, 3024x4032px) Image search: [Google]
IMAG0260.jpg
3MB, 3024x4032px
I have a total 20 mile commute to and from bus stops. I live in Florida and all I have is an old mountain bike that's too small for me.

What can I expect the trade value for the mountain bike? I wanna get a pavement princess that gets me to an from the bus stops and is light weight enough (not looking for carbon fiber) that lifting it on the bike rack on the front of the bus and pedaling an extra 30lbs isn't gonna kill me. I don't know how long I'll have to do this but it's until I can afford to fix my motorcycle (lower main bearing RIP) but I need something to get around on until then.
>>
File: 1396543033585.jpg (48KB, 504x390px) Image search: [Google]
1396543033585.jpg
48KB, 504x390px
my bike is running 100x12 and 142x12 thruaxles.

I had a 15mm thruaxle front wheel lying around and was curious wether I could fit it.
I made a sleve out of aluminium tube with an outside diameter of 15 and inside of 12.
(you can buy similar adapters online)

this part works fine.

however ,I don't get the wheels in my fork.

it seems like the dropouts the selfe are to small.
the endcap won't slot into the fork
how can other people run those adapters ?
is there another hidden standart ?

I even red that the outside diameter of endcaps varies even with the same thruaxle diameter.
>>
>>1100323
You shouldn't be going over 100psi at all, patch or not.
>>
>>1100393
Not true
>>
File: 1231656813342133489121.jpg (3KB, 90x90px) Image search: [Google]
1231656813342133489121.jpg
3KB, 90x90px
>>1100393
Maybe if you stopped being a bum on a walmart BSO you'd know that 100psi is at or below the minimum pressure for most road tires.
>>
>>1100424
You do realise that the rated pressure on the side of the tire isn't actually the best pressure to use?
70-90 is best for 23mm tires, depending on your weight, road quality and weather conditions. Slightly softer tires are better in wet weather. If you're a fatso then sure, pump away.
For 25mm tires, 140psi is suicide madness. for 25mm, 100psi is stupid. In 25mm land, 70psi is good.

Also your back should be inflated more than your front as your back will be holding more weight.
>>
>>1100427
Oh and before you say 'pinch flats', the people who get snakebites are the people who pump their tires up to 110psi in june and then get a pinch flat in november and scream at you for not going up to 110psi
>>
>>1100427
I appreciate moderate pressure as much as the next guy but those numbers seem lower than low unless your last name is quintana or something
>>
>>1100427
>depending on your weight, road quality and weather conditions
Exactly. Saying a certain pressure number is "best" is meaningless.
>>
>>1100424
Minimum pressures are mostly bullshit. 140psi is above the max pressure for most tyres though by about 20psi. Going from 80 to 120psi will gain you 2-3 watts so if you want to sacrifice comfort for that you better be a high level racer. If you actually need 140psi then you're probably quite heavy and rolling on skinny track tyres, in which case you'd probably be running tubulars anyway.
>>
>>1100435
He said a certain range, that's a much better suggestion than just telling people to pump it up to max with no regard to the variables.
>>
File: 146238739789.jpg (48KB, 720x960px) Image search: [Google]
146238739789.jpg
48KB, 720x960px
>>1097622
Is out there a relationship table between tire width and rim width?
sorry about ignorance

pic unrelated
>>
>>1100451
Yes, for wide tyres you want wide rims and narrower rims for skinny tyres. That ensures the tyre is the correct shape when it's inflated and in the case of wider tyres is reduces tyre roll and burping if you run tubeless.
>>
File: ETRTOSchwalbe.jpg (46KB, 600x247px) Image search: [Google]
ETRTOSchwalbe.jpg
46KB, 600x247px
>>1100451
LMGTFY
>>
File: roll.jpg (26KB, 696x398px) Image search: [Google]
roll.jpg
26KB, 696x398px
have anyone used microshift stuff on road/mtb ?
i only use microshit shifters at this moment and they works perfectly on my shimano 8v drivetrain for mtb (22-32-42 & 11-32).
On microshift "catalog" page i saw other components able to be purchased, like cogs and derailleurs.
I mean, against an althus group set, which is more durable? more reliable? well, i would like to read your opinions about this.
>>
>>1100321
you deposit cash to get trial saddle you can try multiple one after an other but not getting the cash back period.
>>
Bike fit question here

So i've recently splashed out on a new bike and have gone on a few nice rides. I matched the fit from my old bike to the dot, every measure i could do.

When cycling i find that my left and only left shoulder muscle lower kind of locks up as if it's cramp, and i have to wave my arm around to relieve the feeling.
I find it weird that it's only that side. It's not that i'm holding onto the bars too hard, or putting weight on that side excessively. And it's not something stupid like the saddle/stem is off at an angle.

Any help greatly appreciated
>>
So the other day my commuter MTB's left crank arm came loose and fell off. Had to bike home with pedal in hand and everything. The next day I found the bolt, and tightened it back on. But every so often I have to retighten it again. Seeing how it's not supposed to come off in the first place, and it's loosening pretty easily, I'm concerned. Loctite didn't do the trick either, so what do /n/? I am missing the little cover that goes around the bolt, but nothing else.
>>
>>1100500
Is it square taper? Over time the square can wear and allow the arm to wiggle on the axle making the bolt come loose. Square taper is shit anyway, time to upgrade to a two piece.
>>
File: x7.jpg (895KB, 2000x3000px) Image search: [Google]
x7.jpg
895KB, 2000x3000px
I'm trying to reassemble a shifter that somebody else disassembled. I can't figure out where the outer end of the spool spring goes. Can anybody help me out? It's a SRAM X7 shifter and I think it's about five years old. I've searched for interior shots of these and found plenty, but I haven't been able to find this exact one and the designs vary enough that that hasn't been helpful.
>>
>>1100507
It is square taper last I checked. Any way to fix it?
>>
>>1100515
Is the part I've circled a hole? If so that looks like the most logical place for it to go to me, hopefully this image makes sense.

>>1100519
If it is wear that's causing it then no, you'd need a new crank arm.
>>
File: jkjkasj.jpg (993KB, 2000x3000px) Image search: [Google]
jkjkasj.jpg
993KB, 2000x3000px
>>1100520
Helps if I post the image.
>>
File: 1977 UO10.jpg (202KB, 1000x702px) Image search: [Google]
1977 UO10.jpg
202KB, 1000x702px
I have an old 70's peugeot 6 speed bike in the process of restoring, should i keep it as a geared bike or convert to a fixie ?
>pic related
>>
File: X7_2.jpg (910KB, 3000x2000px) Image search: [Google]
X7_2.jpg
910KB, 3000x2000px
>>1100521
>>1100520
It is a hole, but it already has something sticking up through it from below, as can be seen more clearly here. The way I have it now doesn't look right to me and it comes out pretty easily, but if I hold the middle of the spring in place with my thumb it does seem to click through all of the gears, so I'm going to assemble it this way and see if it works. This is the third time I thought I might have figured it out, though, so I doubt that I have and if anybody has any other suggestions I still want them. Thanks for your help.
>>
File: jkjkasj1.jpg (992KB, 2000x3000px) Image search: [Google]
jkjkasj1.jpg
992KB, 2000x3000px
>>1100524
The first photo is where the spring naturally rests right, with the shifter in the highest gear (or whatever direction would uncoil the spring)? I guess there doesn't need to be much more tension on the spring so perhaps it goes here, if the circled part is a hole.

This is really bugging me now, wish I could have a look at the thing in person.
>>
>>1100530
Just found this image and it looks like I may be right, unless the spring has popped out and they haven't put it back.

http://forums.mtbr.com/attachments/drivetrain-shifters-derailleurs-cranks/598634d1298249225-2010-sram-x-9-shifter-drops-internal-cable-up-shifts-p1010755-large-.jpg
>>
>>1100522
Fixie fags should be neutered just like their bikes
>>
>>1100530
>The first photo is where the spring naturally rests right, with the shifter in the highest gear (or whatever direction would uncoil the spring)?
Yes.

>I guess there doesn't need to be much more tension on the spring so perhaps it goes here, if the circled part is a hole.
That's what I thought about the tension as well, and the hole you've circled was the first one I tried. Having tried it, I think there needs to be some tension on the spring even when it's in the highest gear because otherwise it doesn't click in to the higher gears as forcefully as the lower ones.

After putting the casing together and putting a cable in it, I can say that the configuration in my second picture results in the best lever feel that I've been able to achieve so far. I won't know for sure whether or not it's right until I get the derailleur connected, but I'm out of time so I'll have to do that later tonight. I'll post the results later on.
>>
>>1100522

Front derailleurs had plastic that cracks with age, so the bike will need that if you keep it geared. The dropouts are narrower than "modern" bikes, so you'll have some fun finding a hub. The 27 inch wheels are becoming problematic too, so if you go with modern 700C aluminum rims you'll need long reach brakes.
>>
>>1100537
It's the spring in the derailer that determines how well it shifts, the spring in the shifter just provides a bit of resistance and allows it to shift without a cable installed (as it needs to be shifted all the way to put the cable in). In theory you could even use it without that spring.
>>
>>1097829
It might be rubbing your cassette somewhere. When my shifting is a little off, fresh lube makes the chain squeak. Try checking the adjustment on your derailer.
>>
File: tyre-pressure.jpg (80KB, 500x275px) Image search: [Google]
tyre-pressure.jpg
80KB, 500x275px
>>1100393
Wrong, see pic.

>>1100328
Get get a set of road tyres for the bike and lock the suspension if it can be locked. I'd also consider looking at the derailleur and cables as well as the brakes to check that they're working nicely. Work with what you got anon, that bike might have good bones for a commuter.
>>
>>1100427
You've never ridden a bike on the track, have you? There's a reason most track pumps are capable of 160 psi.

Also, the pros are running 110-120 a lot of the time and their body weight is from high fifties to hugh sixties.
>>
>>1100494
Are you running drop bars or flat bars? Show a pic if you can.
>>
>>1100571
The pros use tubular tyres. I'm not saying you're wrong but don't they perform differently to clinchers/tubeless setups?
>>
>>1100573
Yes they do, they can hold a lot more pressure. Highest I've seen is 200psi. That other guy is just being pedantic as it's unlikely that a guy running patched tubes is track cycling (and if he is he shouldn't be).
>>
>>1100427
I do t man. I used to run between that 80-90 range. Switched to 105 in front 110 (both 23 and 25mm) in the back and haven't looked back. The bike rolls nicely and smooth.
>>
>you're a big cog
bikeman4UUUUUUU
>>
does anyone know what this white stripe means? these bars are alu
>>
>>1100674
Looks like there used to be a shifter or brake mounted there and it put a little wear on the bar. Nothing to worry about.
>>
>>1100572
Drop bars, yes.
Unable to get a pic unfortunately, nothing untoward though about it that would help... i think
>>
Speaking of tire pressure....

Is there any correlation between flats and tire pressure? Should I keep my tires very high or slightly low if the main goal is no flats? Not too worried about speed/rolling resistance.
>>
>>1100679
Pretty reassuring, thanks anon!
>>
>>1100674
>>1100798
Going by the bar wrap I'm assuming they're drop bars. If the white line is facing forward then it's probably from an interrupter brake lever as those place the cable right up against the bar, then that little dent and ring around the bar would be from the clamp on the lever. As for what caused the white mark, it's either fading from sun exposure or friction from the cable moving about or a combination of both. If it's fading it'll also be the same and the wrap.
>>
>>1100807
>If the white line is facing forward then it's probably from an interrupter brake lever as those place the cable right up against the bar, then that little dent and ring around the bar would be from the clamp on the lever.

That's probably it, I had the same mark on my bars when I had my interrupter levers on it before I swapped out my bars.
>>
File: plank.webm (1MB, 640x360px) Image search: [Google]
plank.webm
1MB, 640x360px
how hard is it to replace cassettes?
>>
>>1097924
>Bar Fly 4 Mini
False: http://cdn.mos.bikeradar.imdserve.com/images/news/2015/02/23/1424735794875-s58jf40vlgfl-1630-354.jpg
>>
>>1100852
If you have a chain whip and a lock ring tool it's very easy. If you don't have a chain whip it's a bit harder as you have to ghetto rig something to hold the cogs, a chain and vice grips will do.
>>
File: wb888888.jpg (209KB, 1217x599px) Image search: [Google]
wb888888.jpg
209KB, 1217x599px
windbreaker question.
i just moved to a new place where i actually have cold weather at points (not even much), and im also just now taking road cycling seriously, training, etc. Thing is i dont know nothing about proper cycling clothes. getting my first windbreaker. have these 2 options. pic related.
pros and cons please, advice or just personal prefference, all helps.

left kind has absovernt inner fabric.. must be way warmer.. but where the hell do i put it when the cold goes away? wont fit in my back pockets right? right one will..the lightweight waterproof kind that usually come in a small bag. do the left one look like its waterproof?

im confused, help me!
thanks! (and sorry for my crap english)
>>
>>1100807
>>1100829
As long as it is not a sign of structural failure! It happened pretty early after I got them, but it had never affected my riding, so I was just wondering..

Thanks again for your input!
>>
>>1100870
Left is a soft shell, not a windbreaker. Soft shells are for when it's actually getting kinda cold, like 40 F or less

If you're dealing with wild temperature swings you're probably going long enough distances that you're a real cyclist and not a dumb ass fred with a minivan following you around. So you would probably have a pannier or a frame bag or something like that. If you don't, get one

Also, consider merino base layers as it can keep you warm when it's cold but when the soft shell is off it lets enough wind through that you stay cool(ish)
>>
>>1100928
thanks a lot, good info.
panniers are kinda a no no due to the type of bike i have (more racing oriented, no where to put one), and also cause i just joined a cycling team with racing objectives. just have a small saddle bar with tools and some food, a couple bucks to buy food sometimes and thats it. water bottles. rides tend to be 40-90 miles, thing is sometimes i go out really early so i have 30-60F oscillations, or worse, im right next to mountains, so if i go climb one, oscillation can be at any time (its cold as hell up there). i guess i can probably tie a soft shell to the waist right? kind of a umb question. and yes..yeah i need to check a first base as they call em here also...dang,so many options, not so much money right now, hah. thanks!
>>
Looking to buy a bike that I can ride in the city - literally only that, that's all it has to do.
I want to spend no more than 800 if possible.

I have absolutely no fucking clue where to buy it, but going off google reviews I found a local store with 500 reviews and like 4,8 average - that has to do it.
Their online shops tells me that all bikes in the price range I mentioned are either from "Kalkhoff" or "Cube".
I am litterally clueless about bikes - does anyone know these brands? Are they any good?
Any tips you can give me so I can make a proper investment here and not waste my money on crap?
>>
First time getting on a bike in decades and I almost ate shit trying to pedal out of the saddle for the first time since I realized I only ever sat. Is it just a matter of git gud?
>>
>>1100980
explain a bit more what you mean with riding on the city. just commuting? how much distance aprox? do you think you may probably like it and ride a bit more than just from A to B?
in any case 800 is more than a decent budget.
also: does any type of bikes call you? mtb/hybrids tend to be solid and comfy as fuck, good for cities where roads arent so perfect or you can go up down curbs etc...more road bike will make you go faster but you need more skill to feel safe... im a fan of buying used cause you can get great deals.. but that means you need to know a bit of bikes..no bike friend near?

my personal choice for cities are semi old hardtail mtb's, this means the ones with only front suspension and rigid frame..they can get very light but can take a beating. if you put thin slick tires (1.5" wide is my fav) and they can fly. you could actually even get a 98-2005 old biike like this for cheap, but so old usually have fucked up drivetrains due to use or just cause their old...so try to get something newer..if used..also cjeck that front suspension fork isnt too heavy so the bike is totally unbalanced, that happens quite a bit....again this would be what i would choose. in anycae you truly could spend less than 800 say 500 if you wanted, but if money isnt a prob... also invest in a good lock so it doesnt get stolen. cube has good bickes, no clue on alkhoff, that may be cause im from overseas, some setuff just doesnt get here.

>>1101004
yeah, just ride a bit more, its incredible how you will get used to the bike and start controlling it better..just ride more and dont get scared. also dont try shit you dont manage too well in busy roads...but go for it!
>>
>>1101029
Alrigh, thanks a lot for your input lad.
I forgot to mention that i am quite heavy, i am 6'4 and around 250 lbs - I am unsure what role that plays but i feel like it does matter.

The reason i ever stopped going to places by bike was because my tires were constantly flat, like every other week or so.
Totally put me off.
But i heared you can get like extra thick tires to work against that.

Distance is only around 12 km total. But yes, if i like it i would probably do it for fun as well, not only to commute.
I will go to the local store tomorrow and see how well the guys there advise - hope they have some knowledge.
>>
>>1101046
if you want write down what the guys offer you, then check some used stuff near your budget zone and then post all the shit here, people will point you out pros and cons of your choices..-- i mean, if you have time and want to.. flats shouldnt be a prob, just buy a foot pump to have at home so you ride the tires at correct pressure. that helps a ton to prevent flats.and yeah, some tires are better than others . good luck, and with your size and experience i would go with a mtb type bike
>>
Replacing FSA Gossamer chainrings with a megatooth for 1x11... are the original bolts gonna be too long?
>>
>>1101111
Probably. If you have the four bolts that thread into the spider then the holes may be deep enough for the bolts to screw in further, or if it's a compact five bolt (so the smaller ring uses a smaller BCD, don't know if that exists in Gossamer but I couldn't find one) then they'll be fine. Bolts are cheap anyway, buy some just in case and if they're not needed at least you'll have spares.
>>
Just rode with drop bars for the first time. Is it supposed to be awkward and tiring at first or did I fuck something up?
>>
>>1101172
Wasn't awkward for me but I did feel it a bit the next day since I wasn't used to going so low.
Maybe it was just your fit but did it hurt being on the hoods too?
>>
>>1101174
Nothing hurt, but my palms felt pretty tired after only half an hour. Maybe my posture is wrong; I might be putting too much weight forward.
>>
>>1101117

Ty, bought some 5mm. Current ones are 7.
>>
I need to upgrade from my old shitty city bike that has a broken front brake and sketchy gears, but I'm on a tight budget and I want something I can use both as a commuter/for fitness and something I can use on forest and gravel roads, is it worth it to buy a cheapo used mountain bike until I can afford a cyclocross and then just repurpose the mountain bike into a winter commuter or something?
>>
my front dérailleur resists jumping gears. i usually ride on the center gear but if i jump or down the chain refuses to move. anyone know what the issue could be?
>>
File: HTB1jYzxSpXXXXb1XVXXq6xXFXXXn.jpg (304KB, 1000x1100px) Image search: [Google]
HTB1jYzxSpXXXXb1XVXXq6xXFXXXn.jpg
304KB, 1000x1100px
Should I go with number 6 or number 12 color?

I already bought a giro synthe from the same seller and it is clearly genuine but I don't like giro helmets, the only helmet I've ever liked is my Prevail so I'm thinking of getting the "fake chinese counterfit that is identical to the real thing for a mere $50 CAD" Prevail 2
>>
>>1101289
12 looks like bananna
>>
>>1101290
I know, it's cool
>>
>>1101291
personally i lke 3 the best
>>
>>1097622

I was wondering if some of you fancy boys could help, I'm looking for a full suspension electric mtb but with throttle, not pedal assist, i don't seem to find anything useful bosch seems to have almost a monopoly on ebikes and all the fun ones are pedal assist. I live in the great EU. Thanks
>>
>>1101311
I'm pretty sure you won't get one with the motor integrated into the frame so you'll be looking at converted bikes. You may struggle to find one though as throttles aren't road legal in the EU which reduces the market for the bikes so shops will have a harder time selling them.

I would suggest trying out some bikes without throttles to see if you really need one, it's less useful with 250w compared to 500w+ (where you might want to run it at reduced power most of the time and then use the throttle for a boost). Newer bikes are also supposedly better at varying the power output based on your pedal input (pedal harder = more motor power), especially the Shimano one.
>>
File: 1500667720019.png (276KB, 380x377px) Image search: [Google]
1500667720019.png
276KB, 380x377px
>>1101316
Fuck, i really just want a throttle one so i can have that option should i choose to overtake or just want acceleration, my body has limits but an electric motor with 60 nm of torque doesn't get tired. If throttle is not allowed then I'll just settle for a regular non e bike, i don't really like the idea of pedal assist no matter how good it is, if i drop 3k euros on an ebike it has to be fast and furious . Fucking bullshit eu fucking fascist faggots with their fucking rules.
>>
>>1101323
Well if you set a throttle-less bike to maximum assistance you won't have to apply any force whilst pedalling as the motor will take over, so unless you're legs are so tired that they literally can't move their own weight it will function similarly to a throttle.
>>
>>1101260
it's either the cable (prob rust or gummed old oil in the housing freezing it in place, could also be it's bolted to the derailleur with too much tension or way loose or not at all)

or it's the limits on the derailleur are set wrong. after the cable is installed, you have to tell the mech how far each direction you want it to travel. this is done via 2 screws, one for either direction, marked L and H. screwed all the way down and the mech won't move at all. too loose, the derailleur cage will pass the intended ring and throw the chain off.

or it's both.

google "sheldon brown derailleur", search keywords in youtube for a visual.
>>
File: bikebags.jpg (291KB, 1024x683px) Image search: [Google]
bikebags.jpg
291KB, 1024x683px
>>1100954
You can get some absolutely huge saddle bags if you need more space to stash your shit. If you want a second bag to stash a layer that isn't in your saddle bag, get a (small) frame bag - they're aero and unobtrusive if done right.

pic related, all the different kinds of sensible non-rack bag
>>
>>1101201

New ones are on the way, but I just machined the original ones down to 5mm at work today. Works perfectly.
>>
File: rack.jpg (149KB, 500x436px) Image search: [Google]
rack.jpg
149KB, 500x436px
So do I have to buy a special kind of bag for this thing or am I supposed to just secure it with bungee cords?
>>
>>1101365
bungees or basket
>>
Are cheap mountain bikes worth buying? I'm looking for something to use as a commuter and on forest trails on a tight budget and I'm trying to look used but I'm 6'3 and I can't find any bikes that fit my size
>>
>>1101311
it's called an electric motorbike you lazy shit
>>
>>1101399
How cheap are you considering?
>>
File: 1500837027740.jpg (95KB, 337x367px) Image search: [Google]
1500837027740.jpg
95KB, 337x367px
How the fuck do you use a crank puller to get your crank off?

I just tried on my bike and instead of the crank coming off all the tool did was strip the threads. Now what do I do?
>>
File: 1502028854565.jpg (135KB, 593x640px) Image search: [Google]
1502028854565.jpg
135KB, 593x640px
>>1101523
Fucking hell I can't even get the fucking headset off without stripping the hex bolt. Just going to pay a bike shop to take my bike apart for restoration.
>>
>>1101523
Same happened to me with a Shimano Octalink crankset, I eventually managed to prise it off with a screwdriver wedged between the arm and bottom bracket and beating on it with a hammer. It would probably be quicker and easier to cut it off though, either split the arm where it wraps around the axle and you may be able to save the bottom bracket, or if you don't mind fitting a new one just cut right through the axle.
>>
>>1101451
As cheap as possible without getting a pile of junk, probably around $4-500
>>
>>1101544
Had a quick look and found these. All come in at the upper end of your budget or slightly more (converting from my currency), anything cheaper isn't really worth the saving.

GT Aggressor Expert/Avalanche Comp/Sport
Cube Aim Pro
Commencal El Camino
>>
>>1101544
best value is the used market but may take some time to find your size. find the /bbg/ and post according to the instructions in the OP
>>
File: flag_yellow_low.jpg (54KB, 600x401px) Image search: [Google]
flag_yellow_low.jpg
54KB, 600x401px
does that represent bearings
>>
Do I need tubeless rims or is ghetto tubeless for MTB pretty well sorted these days?

My rims are just Merida factory rims- Alex maybe- and while I'd be happy to upgrade when I finally kill the fuckers, I don't want to splurge for no good reason.
>>
>>1101618
Depends how well the beads lock in place, if at all. Pumps your tyres up nice and hard, like 60-70psi, and see if you get any pinging and can see the bead popping into place. Then let all the air out and see if the beads stay in place, you want it so you have to apply a bit of pressure to push them into the centre of the rim.
>>
>>1101619

Wait- so no tube, no sealant, factory tape and pump th up to max psi to see what happens or splurge on setting them up and if it fails just bang some tubes back in?
>>
File: cotterless2sm.jpg (22KB, 500x356px) Image search: [Google]
cotterless2sm.jpg
22KB, 500x356px
>>1101523
Did you remember to remove that locknut that screws into the end of the axle?
>>
>>1101620
No, do it with the tube, you're just checking how well the beads seat.
>>
File: x.png (2MB, 1598x600px) Image search: [Google]
x.png
2MB, 1598x600px
Are these guys still in good condition? Didn't take the photos myself, was gonna buy a new casette tomorrow but they don't look all that bad. The owner doesn't know shit about bikes, she says that the sprockets are loose/wobbly. I don't want to make her pay for a new casette if I can just fix it by reassembly.

tl;dr: should these be replaced?
>>
>>1101791
They just look dirty. Nothing on the teeth would suggest wear, though.
>>
I want to get a bike so I can ride to my local library. I intend to stay in the bike lane / road but not all roads are great in Tucson and I might need to ride on the dirt once in a while. What kind of bike should I get? Also do I need special wheels for hot climates? It's over 100F every day for months during the summer. I think that might have an effect on the wear and tear of the bike.
>>
>>1101852
I'm assuming Tucson is dry. hot and dry might cause tires to crack faster than normal, maybe rubber brake pads too. otherwise moisture, rain and especially road salt will have a much stronger effect on all the metal and bearings which you will probably have none of.
clean the dust out of the chain and try not to store the bike in the sun. if you get a bike with a suspension fork, clean the dust off the fork stanchions.
the 100F may instead wear you out. I can barely ride over 85. wear climate appropriate helmet and clothes.
>>
>>1101864
Hey thanks for the reply! I kinda need to know what kind of bike, do I get the one with the really thin wheels or like a mountain bike? I know there are also hybrid ones but I'm pretty lost. I see people riding in the street with all kinds. We have good bike lanes but there is a lot of dirt sometimes.
>>
>>1101867
"not all roads are great" is very subjective.
i dont know your budget
i dont know your size
did you ever have a bike before? how much experience do you have?
are you handy? are you afraid to wrench on it?

completely blind advice would be to get an old name brand mountain bike with no suspension if you aren't afraid to work on it from day one and maybe replace a few parts.
for example this: https://tucson.craigslist.org/bik/d/trek-7000/6190536775.html
i would clean and tune the drivetrain, inspect the brakes, maybe change the pads, and get semi slick tires.

if you have some extra cash maybe a low end mountain or hybrid from a big name brand.
I think road bikes are less forgiving to novices, both in comfort and handling.

get it from a local bike store as it would be assembled by hopefully someone experienced. the shops also often have free service for a year or so on new bikes.
you can also try out all different bikes there and they will help you out with sizing and fit.
>>
>>1101871
Didn't know owning a bike was so involved. I had a bike growing up but I never had any trouble with it aside from the odd flat tire. So I guess I got pick up a mountain bike from a local store?
>>
>>1101872
its involved if you ride a lot and wear it out. or if you buy a used bike (a.k.a. someone else's problem)

comfort is vital. id rather ride a mediocre bike set up properly for my dimensions than a top of the line bike that';s way out of my dimensions.

go to a local shop and try different ones out
>>
>>1101876
Alright thanks for the advice. Can you recommend certain brands/models? Like a go to reliable thing.
>>
>>1101878
personal anecdote; my dad bought me a Trek 3700 when I was 13 (i believe it was $200 at the time). the sole reason I chose it was because of its "mega drive" super low first gear. I rode it hard and left out in the rain sometimes and so forth. I only changed brake pads and tubes until I was 20. then I started to actually take care of it and fix it up, new saddle, tires, rear wheel, pedals, chain, cassette, derailleur, crank, cables, other bits and bobs. I got it to ride almost good as new when I was 23. (the suspension fork was neglected beyond repair)
then it was stolen. if it wasn't stolen i would still ride and both my younger brothers would still be riding it daily.

i am of the opinion the cheapest adult sized bike from a big name brand is better than any walmart nonsense, or most bikes from bikesdirect.com
>>
>>1101887
Will keep that in mind, thanks.
>>
Any of you guys get cramps around the ribs/lung area? Never had them before until I started to push a bit harder now. Suggestions?
>>
File: 1410114661145.jpg (27KB, 419x566px) Image search: [Google]
1410114661145.jpg
27KB, 419x566px
>>1101838
Aight, thanks for the second opinion. I'll clean her up good.
>>
>>1101913
You mean like stitches or actual cramps in your diaphragm? You may be constricting your torso too much if you're hanging in the drops (I assume roadbike). Try increasing your output slowly and steadily and see if they go away, your body has to get used to so much more workload if you just go hard in the paint all of a sudden.
>>
>>1101916
Thanks, yes cramps. It does slowly go away when I drink water and stretch to an upright position.
>>
File: frame.jpg (122KB, 1293x621px) Image search: [Google]
frame.jpg
122KB, 1293x621px
how difficult is it to build your own bike from a chinese carbon frame?
>>
How do I learn to do sick jumps without embarrassing myself? I'm quick down the flow track, have pretty good balance, but haven't jumped a bike since I was a child on a bmx. Can I just buy a plastic skate ramp and practice launching off it in the backyard?
>>
My front gears won't shift all the time. Is it an easy fix for a fred?
>>
>>1102629
Yeah, just pull them off.
>>
>>1102629
Downshift your front mech. Loosen the cable clamp, most likely with a 5mm allen. Pull the cable taut with some pliers, and retighten the clamp.

Also make sure your high limit screw is set properly, it might be just a little too far in.
>>
Is using a derailleur as a chain tensioner okay? I removed all the cables and tuned it with the little screws. It seems to work okay. Any reason not to do this?
>>
>>1102646
is the limit screw the one the guy at the shop told me to never touch? also thanks.
>>
>>1102870
Yes. If they're set up right, you should never have to touch them again. If your shifting used to work right, then it's probably just that your cable has stretched out, which is normal.

Also, I said the wrong thing, it's probably a 4mm allen, not a 5.
>>
>>1102753
Sheldown Brown said it was ok

https://www.sheldonbrown.com/singlespeed.html

>Recycled Rear Derailers

>You can also use your old rear derailer as a chain tensioner, but it is heavier than you need, introduces more friction, and detracts from the looks of the bike. You need to adjust the high-gear limit stop for the correct chainline.

They're not that expensive so it's up to you, should be fine.
>>
>>1102906
Sheldon Brown

oh boy, do I feel terrible for that typo. RIP Sheldon
>>
How do I remove the headset hex bolt from an old bike?

It just wont budge, it's chewing up my tools and/or it'self rather than move.
>>
So I'm sitting next to my bike and suddenly hear a rush of air leaking out of it. I could feel the air as I held my hand over the valve and I see the rubber on the side of the valve has a little cut.

Can I repair this to use the tube again? I already replaced it but I was wondering whether or not to just use the tube for other stuff.
Thread posts: 378
Thread images: 70


[Boards: 3 / a / aco / adv / an / asp / b / bant / biz / c / can / cgl / ck / cm / co / cock / d / diy / e / fa / fap / fit / fitlit / g / gd / gif / h / hc / his / hm / hr / i / ic / int / jp / k / lgbt / lit / m / mlp / mlpol / mo / mtv / mu / n / news / o / out / outsoc / p / po / pol / qa / qst / r / r9k / s / s4s / sci / soc / sp / spa / t / tg / toy / trash / trv / tv / u / v / vg / vint / vip / vp / vr / w / wg / wsg / wsr / x / y] [Search | Top | Home]

I'm aware that Imgur.com will stop allowing adult images since 15th of May. I'm taking actions to backup as much data as possible.
Read more on this topic here - https://archived.moe/talk/thread/1694/


If you need a post removed click on it's [Report] button and follow the instruction.
DMCA Content Takedown via dmca.com
All images are hosted on imgur.com.
If you like this website please support us by donating with Bitcoins at 16mKtbZiwW52BLkibtCr8jUg2KVUMTxVQ5
All trademarks and copyrights on this page are owned by their respective parties.
Images uploaded are the responsibility of the Poster. Comments are owned by the Poster.
This is a 4chan archive - all of the content originated from that site.
This means that RandomArchive shows their content, archived.
If you need information for a Poster - contact them.